Integral calculus: plane areas in rectangular coordinates

Click For Summary
The area between the curve y = 1/(x^2 + 1) and the x-axis from x = 0 to x = 1 is calculated using integral calculus. The solution involves determining the height of the curve at the endpoints, where y = 1 at x = 0 and y = 1/2 at x = 1. The area is expressed as A = ∫ from 0 to 1 (1/(x^2 + 1)) dx, which evaluates to A = π/4 square units. The steps taken to arrive at this solution are confirmed to be correct. The process demonstrates the application of integral calculus to find areas under curves.
delapcsoncruz
Messages
20
Reaction score
0

Homework Statement



Find the area between y= 1/(x2+1) and the x-axis, from x=0 to x=1


The Attempt at a Solution



so when x=0, y=1
and when x=1, y=1/2

next i plot the points, so the intersection of the given equation is (0,1) and (1,1/2)
Yh= Y-higher= 1/(x2+1)
Yl= Y-lower= 0
the strip is vertical, so the length (L) = (Yh-Yl) and the width (W) is dx


dA=LW
dA=(Yh-Yl)dx
dA=(1/(x2+1))dx
A=∫from 0-1 dx/(x2+1)
A=Arctan x from 0-1
A=Arctan 1 -Arctan 0
A=pi/4 sq.units


was my solution correct?
 
Physics news on Phys.org
looks good to me
 
delapcsoncruz said:

Homework Statement



Find the area between y= 1/(x2+1) and the x-axis, from x=0 to x=1

The Attempt at a Solution



so when x=0, y=1
and when x=1, y=1/2

next i plot the points, so the intersection of the given equation is (0,1) and (1,1/2)
Yh= Y-higher= 1/(x2+1)
Yl= Y-lower= 0
the strip is vertical, so the length (L) = (Yh-Yl) and the width (W) is dx

dA=LW
dA=(Yh-Yl)dx
dA=(1/(x2+1))dx
A=∫from 0-1 dx/(x2+1)
A=Arctan x from 0-1
A=Arctan 1 -Arctan 0
A=pi/4 sq.units

was my solution correct?
Yes.

You went through a lot of steps to get the answer.
 
Question: A clock's minute hand has length 4 and its hour hand has length 3. What is the distance between the tips at the moment when it is increasing most rapidly?(Putnam Exam Question) Answer: Making assumption that both the hands moves at constant angular velocities, the answer is ## \sqrt{7} .## But don't you think this assumption is somewhat doubtful and wrong?

Similar threads

Replies
3
Views
2K
  • · Replies 10 ·
Replies
10
Views
2K
  • · Replies 5 ·
Replies
5
Views
2K
  • · Replies 3 ·
Replies
3
Views
1K
  • · Replies 14 ·
Replies
14
Views
2K
  • · Replies 105 ·
4
Replies
105
Views
6K
  • · Replies 9 ·
Replies
9
Views
2K
  • · Replies 4 ·
Replies
4
Views
2K
  • · Replies 13 ·
Replies
13
Views
1K
  • · Replies 1 ·
Replies
1
Views
2K